ChaseDream
搜索
12下一页
返回列表 发新帖
查看: 5597|回复: 15
打印 上一主题 下一主题

请教LSAT-9-I-10,11

[复制链接]
楼主
发表于 2003-6-2 17:46:00 | 只看该作者

请教LSAT-9-I-10,11

Questions 10-11

If the public library shared by the adjacent towns of Redville and Glenwood were relocated from the library’s current, overcrowded building in central Redville to a larger, available building in central Glenwood, the library would then be within walking distance of a larger number of library users. That is because there are many more people living in central Glenwood than in central Redville, and people generally will walk to the library only if it is located close to their homes.

10 Which one of the following, if true, most strengthens the argument?

(A) The public library was located between Gienwood and Redville before being moved to its current location in central Redville.

(B) The area covered by central Glenwood is approximately the same size as that covered by central Redville.

(C) The building that is available in Glenwood is smaller than an alternative building that is available in Redville.

(D) Many of the people who use the public library do not live in either Glenwood or Redville.

(E) The distance that people currently walk to get to the library is farther than what is generally considered walking distance.

答案:B, 我不明白B为什么是加强了,是无关啊?不过,其它几个选项都不行.

11. Which one of the following, if true, most seriously weakens the argument?

(A) Many more people who currently walk to the library live in central Redville than in central Glenwood.

(B) The number of people living in central Glenwood who would use the library if it were located there is smaller than the number of people living in central Redville who currently use the library.

(C) The number of people using the public library would continue to increase steadily if the library were moved to Glenwood.

(D) Most of the people who currently either drive to the library or take public transportation to reach it would continue to do so if the library were moved to central Glenwood.

(E) Most of the people who currently walk to the library would remain library users if the library were relocated to central Glenwood.

答案:B,请问A,B有什么区别吗?都行啊.都是削弱.

谢谢.
沙发
发表于 2003-6-3 00:42:00 | 只看该作者
我不知道我说清楚了没有。如果你觉得我说的不清楚,请指出不足之处。

10。我觉得B是这样加强的。
这个argument的结论是:the library would then be within walking distance of a larger number of library users。就是说图书馆将在更加多的人的步行范围内。
而这个argument的原因是:there are many more people living in central Glenwood than in central Redville(更多的人住在的central Glenwood 比起central Redville)。
这里并没有很清楚地说明一个城市的central区域是否within walking distance of a larger number of library users? 如果Glenwood 比Redville大上很多倍,如那central Glenwood 比起central Redville的面积要大,那就不可能导致结论了。所以B就可以说是加强。


11。A不是削弱。A说现Many more people who currently walk to the library live in central Redville than in central Glenwood. (现在更多住在central Redville的人走路去library比起住在central Glenwood的人), 与作者的argument没有冲突。完全有可能因为作者所说的原因,而推出作者所说的结果。就是说:即使现在更多住在central Redville的人走路去library比起住在central Glenwood的人,但是如果library从central  Redville搬到central  Glenwood后,因为central  Glenwood的人多,而导致the library would then be within walking distance of a larger number of library users。

板凳
 楼主| 发表于 2003-6-3 12:39:00 | 只看该作者
非常感谢wdzc!!!!

你刚考完试,多休息几天吧.

你讲的T10题我明白了,T11题,还是没明白.转糊涂了.

谢谢.
地板
发表于 2003-6-3 14:09:00 | 只看该作者
10)排除一个一个不利于推理情况。
11)A翻译成:现在步行去图书馆的人中,住在Redville城市中心的比住在Glenwood城市中心的人多。这个只是描述搬迁之前的情况,但不能说明或推出搬迁之后的情况,与结论无关。
5#
 楼主| 发表于 2003-6-4 13:50:00 | 只看该作者
第11题,还是糊涂着呢。

能不能再讲讲?谢谢。
6#
 楼主| 发表于 2003-6-4 21:36:00 | 只看该作者
以下是引用joywzy在2003-6-4 13:50:00的发言:
第11题,还是糊涂着呢。

能不能再讲讲?谢谢。


能否帮助再讲讲这一题?谢谢。
7#
发表于 2003-6-5 09:40:00 | 只看该作者
11) 结论是:如果搬迁图书馆,在步行去图书馆范围内的用户会增加
A翻译成:现在步行去图书馆的人中,住在Redville城市中心的比住在Glenwood城市中心的人多。这个只是描述搬迁之前的情况,但不能说明或推出搬迁之后的情况,与结论无关。
也就是A谈论搬迁之前的情况,对搬迁之后,不知道住在Glenwood城市中心的人步行去图书馆范围内的用户是否会增加。换句话说,搬迁之后A是否能成立不知道。
B:如果图书馆在G中心,那么住在G中心要使用图书馆的人要比少于 图书馆在R中心时的R城市中心的用户。并不是直接weaken 结论。因为对步行距离不知道。但是 weaken是一种效果,对结论要产生不利影响。
对weaken/strength 题,答案是排除出来的。对所有答案进行比较找出效果最强的一个。
8#
发表于 2003-6-5 11:14:00 | 只看该作者
I am not helping here, because I holds a difference opinion to the answer to question 11, and I need help!

I would like to pick choice D.

It seems to me the whole argument is concerned about "if relocate the library to the central of Glewood would increase the number of people who will walk to library, due to the fact that the library will be within a walking distance, plus there are more people lives in the central of Glewood than in the central of Redville. "

And choice D shows the relocation of the library will not change the choice of transportation by car / public transportation to by walk.

I think this is somehow weaken the argument.

Please help!!!/   tks
9#
发表于 2003-6-5 18:34:00 | 只看该作者
Let me try NO11,

原文的推理有一个逻辑漏洞。其推理过程为:

because: (1)住在G图书馆步行范围内的人>住在R图书馆步行范围内的人+ (2)people generally will walk to the library only if its is located close to their homes.

therefore: G步行范围内用图书馆的人>R步行范围内用图书馆的人(the library would then be within walking distance of a larger number of library users.)

注意第二个条件,only if 后跟的是必要条件而不是充分条件,也就是people walk to the library→ library locate close to their homes. 不是:“只要图书馆离家近,人们就去图书馆”;而是:“如果人们去图书馆,那么图书馆离他们家近”。

所以weaken选项B指出,G“住”在离图书馆步行范围人的人虽多,但“用”图书馆的人却不一定比R多。注意条件里“住”与结论里“用”这两个概念之间的区别,B项即是在这两个概念之间断桥。

至于A项,同意楼上的说法是无关项,因为讲的是现在走着去R图书馆的人,更多地住在R。这个条件与原文推理中的两个条件都没有关系,所以是无关项。

之所以选D项,我觉得是没有搞清结论说的是什么。结论只说“步行”去用图书馆的人会增多,与坐车去的没关系。

不知我理解得对不对,请大家指教!

10#
发表于 2003-6-6 07:43:00 | 只看该作者
I agree choice B is the best one to weaken the argument. And choice D (I think, could be able to weaken the topic too, but just not so good as choice B), or maybe it goes too far.

Since we need the strongest one, choice B is good!

thank you so much!! very helpful.
您需要登录后才可以回帖 登录 | 立即注册

Mark一下! 看一下! 顶楼主! 感谢分享! 快速回复:

手机版|ChaseDream|GMT+8, 2024-10-2 06:10
京公网安备11010202008513号 京ICP证101109号 京ICP备12012021号

ChaseDream 论坛

© 2003-2023 ChaseDream.com. All Rights Reserved.

返回顶部